Is There an Integration Factor for This Differential Equation?

Click For Summary
The discussion revolves around solving the differential equation cos(t) dv/dt + sin(t) t = (GM/b^2) sin^3(t). Participants express difficulty in identifying an appropriate integration factor and suggest that the left-hand side resembles a product rule with an incorrect sign. A substitution of x = sin(t) is proposed as a potential solution strategy. Additionally, there is a suggestion that the original equation may have been misstated, possibly involving a term with v instead of t. The conversation highlights the complexities of integrating the terms involved.
binbagsss
Messages
1,291
Reaction score
12

Homework Statement


##cos t \frac{dv}{dt} + (sin t) t = \frac{GM}{b^2 }\sin^3 t ##

Homework Equations



above

The Attempt at a Solution



im pretty stuck to be honest. It almost looks like a product rule on the LHS but it has the wrong sign, RHS I've tried writing ##sin^3 t## as ##(1-cos^2t)\sin t## etc, pretty unsure what integration factor I need.

Thanks in advcance.
 
Physics news on Phys.org
Hi binaqsss:

I suggest trying the substitution x = sin(t).

Regards,
Buzz
 
binbagsss said:

Homework Statement


##cos t \frac{dv}{dt} + (sin t) t = \frac{GM}{b^2 }\sin^3 t ##

Homework Equations



above

The Attempt at a Solution



im pretty stuck to be honest. It almost looks like a product rule on the LHS but it has the wrong sign, RHS I've tried writing ##sin^3 t## as ##(1-cos^2t)\sin t## etc, pretty unsure what integration factor I need.

Thanks in advcance.

If you wrote the problem correctly, then you just have
$$\frac{dv}{dt} = - t \tan t + a \frac{ \sin^3 t }{\cos t} = - t \tan t + a \sin^2 t \, \tan t,$$
so ##v = - \int t \tan t \, dt + a \int sin^2 t \, \tan t \, dt##. The integration in the first term involves some non-elementary functions.
 
binbagsss said:

Homework Statement


##cos t \frac{dv}{dt} + (sin t) t = \frac{GM}{b^2 }\sin^3 t ##
Did you perhaps mistype it and mean instead:$$
\cos t \frac{dv}{dt} + (\sin t) \color{red}{v} = \frac{GM}{b^2 }\sin^3t \text{ ?}$$
 
Question: A clock's minute hand has length 4 and its hour hand has length 3. What is the distance between the tips at the moment when it is increasing most rapidly?(Putnam Exam Question) Answer: Making assumption that both the hands moves at constant angular velocities, the answer is ## \sqrt{7} .## But don't you think this assumption is somewhat doubtful and wrong?

Similar threads

  • · Replies 5 ·
Replies
5
Views
2K
  • · Replies 1 ·
Replies
1
Views
2K
  • · Replies 3 ·
Replies
3
Views
2K
  • · Replies 22 ·
Replies
22
Views
3K
  • · Replies 2 ·
Replies
2
Views
2K
Replies
8
Views
1K
Replies
3
Views
2K
  • · Replies 1 ·
Replies
1
Views
1K
Replies
7
Views
2K
Replies
2
Views
1K